Sie sind auf Seite 1von 5

Data Sufficiency Question Bank - Practice Questions

A data sufficiency question has a question followed by two statements. You will be expected to answer if the information provided in the statements is sufficient to answer the question. You answer will be

1.
2. 3. 4.

Choice 1 if the question can be answered using one of the statements alone, while the other statement is not sufficient to answer the question. Choice 2 if the question can be answered using each of the statements independently Choice 3 if both the statements together are needed to answer the question Choice 4 if both the statements independently or taken together are not sufficient to answer the question

A collection of questions that typically appear from the topic of data sufficiency. Question 1: How long will it take for two pipes A and B to fill an empty cistern if they worked alternately for an hour each? (A) Working alone, Pipe A can fill the cistern in 40 hours (B) Pipe B is one third as efficient as Pipe A

Correct choice (3). Correct Answer - (Both the statements together are sufficient to answer the question.) Explanatory Answer: From statement A, we know that Pipe A can fill the tank in 40 hours. However, this information is not sufficient as we do not have the data for Pipe B. Hence, statement A alone cannot answer the given question. From statement B, we know that Pipe B is one third as efficient as pipe A. However, we do not know the rate at which Pipe A fills the tank. Hence, we will not be able to find the rate at which Pipe B fills the cistern. Therefore, statement B alone is not sufficient to answer the question. Now, if we combine the two statements, we know that Pipe A take 40 hours to fill the cistern. Pipe B takes 120 hours to fill the cistern. If they worked alternately, then either Pipe A could have started the cycle or Pipe B could have started the cycle. If Pipe A started the sequence of filling alternately, then at the end of two hours, the two pipes together would have filled will fill in 30 hours. th of the tank in an hour. Or the cistern

If Pipe B started the sequence, then at the end of 2 hours, the two pipes together would have filled th of the tank in an hour. Or the cistern will fill in 30 hours.

As the answer obtained irrespective of which pipe started the sequence is the same, the correct answer is (3) - i.e., both the statement are required to answer the question.

Question 2: What is the value of X, if X and Y are two distinct integers and their product is 30?

(A) X is an odd integer (B) X > Y

Correct choice (4). Correct Answer - (The value of X cannot be determined from the information provided)

Solution: From the question, we know that both X and Y are distinct integers and their product is 30. 30 can be obtained as a product of two 1 * 30 2 * 15 3 * 10 5 * 6 distinct integers in the following manner (-1) * (-30) (-2) * (-15) (-3) * (-10) (-5) * (-6)

Statement I: From this statement, we know that the value of X is odd. Therefore, X can be one of the following values: 1, -1, 3, -3, 5, -5. So, using the information in statement I we will not be able to conclusively decide the value of X. Hence, statement I alone is not sufficient to answer the question. Statement II: From this statement, we know that the value of X > Y. From the given combinations, X can take more than one value. Hence, using the information in statement II, we will not be able to find the value of X. Combining the two statements, we know that X is odd and that the value of X > Y. The combinations that satisfy both the conditions include X taking the value of -1, -3 and 5. As the information provided in the two statements independently or together are not sufficient to answer the question, the answer choice is (4).

Question 3: The set S of numbers has the following properties: I) If x is in S, then 1/x is in S. II) If both x and y are in S, then so is x + y. Is 3 in S? (A) 1/3 is in S. (B) 1 is in S.

Correct Answer - (2) Solution:

Consider (1) alone. Since 1/3 is in S, we know from Property I that 1/(1/3) = 3 is in S. Hence, (1) is sufficient.

Consider (2) alone. Since 1 is in S, we know from Property II that 1 + 1 = 2 (Note, nothing in Property II prevents x and y from standing for the same number. In this case both stand for 1.) is in S. Applying Proper ty II again shows that 1 + 2 = 3 is in S. Hence, (2) is also sufficient.

Question 4: Is x = y?

(A) (B) (x

=4 50) = (y
2

50)

Correct Answer - (1) Solution:

Statement

A.

=>

=>

2.

This is

possible when

y.

Hence

statement

alone

is sufficient.

Statement B. (x 50)2 = (y 50)2 . We cannot say if x = y in this case. Take for example, let x = 100 and y = 0. Then (x 50)2 = (100 50)2 = 502 = 2500. And (y 50)2 = (0 50)2 = 502 = 2500.

Hence, statement A alone is sufficient and statement B alone is not sufficient.

Question 5: Is the smallest of five consecutive integers even? (A) The product of the five integers is 0 (B) The arithmetic mean of the five integers is 0.

Correct Answer - (1) Solution:

If the smallest of five consecutive integers is even, then the first, third and fifth integers will be even. From statement A, we know that one of the 5 numbers is 0. However, we will not be able to say which of the 5 numbers happen to be 0. From statement B, we know the arithmetic mean of the 5 numbers is 0. The A.M of five consecutive integers is the third integer, which is 0. 0 is even. Hence, the smallest of the 5 consecutive integers is even. Hence statement B alone is sufficient and the answer is (1).

Question 6: Is X a prime number, given that X is a positive integer? (A) X > 3000 4 (B) X < 10,000
4

Correct Answer - (3) Solution:

From statement A we know that X4 is greater than 3000. There are infinite values for X possible for which X4 is greater than 3000 From statement B we know that X 4 is less than 10,000. This one is a lot better. There are only 9 integer values that satisfy this condition. But this still does not give us a unique answer. Combining the two statements, we know that 2000 < X4 < 10000. The following values of X satisfy this condition -> 8 and 9. Both 8 and 9 are not prime. Hence, the question can be answered conclusively using the two statements. Hence answer choice (3)

Question 7: Is m divisible by 6? (A) m is divisible by 3 (B) m is divisible by 4

Correct choice - (3)

Explanatory answer We need to answer if m is divisible by 6. The answer has to be a definitive YES or a NO. The test of divisibility for 6 is that the number should be divisible by both 3 and 2. From statement (A) we know that m is divisible by 3. However, this does not answer the question if m is also divisible by 2. Hence, statement (A) alone is not sufficient. We can ru le out answer choices (2). The correct answer has to be between (1), (3) or (4). From statement (B) we know that m is divisibly by 4. If m is divisible by 4, then m should surely be divisible by 2. However, from statement (B) alone we do not know if m is divisible by 3. Therefore, statement (B) alone is also not sufficient. Hence, we can eliminate answer choice (1). Combining the two statements, we know that m is divisible by 3 and by 4. Hence, we can conclude that m is divisible by 6. Choice (3 ) is correct.

Question 8: If a salesman received a commission of 3% of the sales that he has booked in a month, what was the sales booked by the salesman in the month of November 2003?

Correct choice - (2)

Explanatory answer From statement (1), we know that the sales value after the salesmans commission. If his commission is 3% of the sales booked. Then the net sales value is 100 3 = 97% of the sales booked. From statement (1), we know that 97% of sales booked = Rs.245,000. So we can find out the sales booked. Statement (1) alone is sufficient. From statement (2), we know that the original cost of the products is Rs.225,000. We know the sales booked = 1.25 * 225,000. Hence, statement (2) is also sufficient. As each of the two statements are independently sufficient to answer the question, choice (2) is the best answer.

Das könnte Ihnen auch gefallen